Sunteți pe pagina 1din 81

Prelims 2017 – Test 31

SOLUTIONS

Legend: Elimination based questions are star-marked in explanations.

1. The railways has started giving accident insurance to such


passengers who book their tickets through online mode. Consider
the following with reference to it.
1. Citizens of foreign countries are not eligible for this scheme.
2. Presently, an insurance premium of ten rupees is charged by the
railways per ticket.
3. Such insurance is to be readily provided at no additional cost for
children below five years of age.
4. The insurance coverage is uniform for all cases of injury and
death being pegged at one lakh rupees.
Select the correct answer using the codes below.
a) 2, 3 and 4 only
b) 3 and 4 only
c) 1 only
d) 1 and 2 only
Solution: c)
Justification: Statement 1: The scheme is applicable only for Indian
Citizens who book their e-ticket through NGeT Website Application only.
Citizen of foreign countries are not eligible.
Statement 2: Initially, the premium was Rs. 92 paise per passenger
inclusive of all taxes, but the scheme is free of cost now.
Statement 3: The travel insurance will not be provided for the children
below 5 years of age as per rules.
Statement 4: The Travel Insurance Scheme shall be kept uniform for all
classes but differ according to the case:
The coverage for Hospitalization Expenses for Injury is over and above
the death/permanent total disability/partial disability.
Q Source: http://contents.irctc.co.in/en/InsuranceTermCondition.pdf

2. Money market is a market for


a) Long term borrowing for a period of 3 years or more
b) Initial Public Offers (IPOs) of Public sector companies
c) Short-term borrowing and lending
d) Foreign securities and bonds
Solution: c)
Learning: The money market is where financial instruments with high
liquidity and very short maturities are traded.
Maturities of instruments usually range from overnight to just under a
year.
They help industries secure short-term loans to meet their working
capital requirements through the system of finance bills, commercial
papers, etc.
Capital market provides long term source of funding.
Q Source: Indian Economy: Ramesh Singh

3. Consider the following:


1. Commonwealth of Nations
2. BIMSTEC
3. Asia-Pacific Economic Cooperation
4. Shanghai Cooperation Organisation
India is a member of which of the above?
a) 1 and 2 only
b) 2, 3 and 4 only
c) 1, 2 and 4 only
d) 1, 3 and 4 only
Solution: c)
Learning: Among major organizations, India is a member of the
Commonwealth of Nations, the South Asian Association for Regional
Cooperation, BIMSTEC, the Non Aligned Movement, the G20, the G8+5.
Others include the International Monetary Fund, the World Bank, the
World Trade Organisation, the United Nations, the BRICS, the Shanghai
Cooperation Organisation, the Asian Infrastructure Investment Bank
and Missile Technology Control Regime.
India has recently joined the MTCR, SCO and AIIB.
Q Source: India and International organizations

4. Communications signals can be sent either by analog signals or


digital signals. What is the benefit of sending signals via digital
channels?
1. It can maintain signal quality over longer distances than
analogue.
2. It carries lesser information per second making it less
vulnerable to hacking.
Which of the above is/are correct?
a) 1 only
b) 2 only
c) Both 1 and 2
d) None
Solution: a)
Justification: For an analog signal, the signal is varied continuously
with respect to the information. In a digital signal, the information is
encoded as a set of discrete values (for example, a set of ones and zeros).
Statement 1: During the propagation and reception, the information
contained in analog signals will inevitably be degraded by undesirable
physical noise.
Digital signals maintain their quality over long distances better than
analogue signals because additive noise can be easily substracted from
the discrete value signals.
Statement 2: Digital signals carry more information per second than
analogue signals. This is the same whether optical fibres, cables or radio
waves are used.
Q Source: Basics of Telecommunication

5. In context of World History, what was referred to as “Iron


Curtain”?
a) Economic growth of iron ore rich and iron ore poor nations
b) Development divide between colonized and colonizer nations
c) Contrast in the progress of Industrial revolution in the East and
West
d) Boundary that divided communist and capitalist states in Europe
Solution: d)
Learning: The term Iron Curtain refers to the boundary that divided
Europe in the west and the Soviet Union and its Communist one-party
states in the east.
The division began at the end of World War Two in 1945 and lasted until
the fall of the USSR in 1989.
It was a political, military, and ideological barrier erected by the Soviet
Union; a fall out of the cold war.
Q Source: Improvisation: 12th Contemporary World Politics NCERT

6. Solibacillus ‘Kalamii’ was recently in news due to which of the


following reason(s)?
a) It was discovered on the International Space Station (ISS).
b) It does not have a metabolism of its own.
c) It was the first bacteria to be found on Mars.
d) Both (a) and (c)
Solution: a)
*Justification*: You can eliminate B as bacteria have metabolism of
their own. Option C can be eliminated as no live signs of life have been
found on Mars.
Researchers found that the bacteria remained on board the International
Space Station for 40 months. The new organism is found only on the ISS
and has not been found anywhere on the earth.
It was named by NASA Solibacillus kalamii to honour the late former
president of India Dr A P J Abdul Kalam.
Q Source: http://www.thehindu.com/news/international/from-nasa-
with-love-new-bacteria-named-after-kalam/article18518884.ece

7. Consider the following:


Mild Hybrid Vehicles
1. Run entirely on water as a fuel
2. Assist normal engine save fuel by using recovered energy stored
in a battery
3. Do not generate any carbon based emissions
Which of the above is/are correct in this reference?
a) 1 only
b) 2 and 3 only
c) 2 only
d) 1, 2 and 3
Solution: c)
Justification: Mild hybrid technology is different from plug-in or
conventional hybrid vehicles.
• Mild hybrid vehicles use the energy generated while applying the
brakes and turns it into electric energy that is then stored in a
battery.
• This energy can then be used to turn the starter motor when the
car’s start stop system needs it.
• In most cases, a mild hybrid will turn its engine off when at idle or
when it comes to a halt at a traffic signal for instance.
• Research shows that mild-hybrids are only 7-15% more fuel
efficient as compared to conventional hybrid systems that can run
on pure electric power for short distances too.
Q Source:
http://timesofindia.indiatimes.com/auto/miscellaneous/subsidy-for-
mild-hybrid-vehicles-under-fame-scheme-
removed/articleshow/57982359.cms

8. Recently a coordinated global cyberattack known as WannaCry was


conducted. It was a ransomware and could have affected the cyber
security of computers by
1. Setting all optical fibre internet lines of Internet Service
Providers (ISPs) to simultaneous network failure by beaming
internet back to source
2. Encrypting essential files on your device and requiring that you
pay an amount to unlock those files
Which of the above is/are correct?
a) 1 only
b) 2 only
c) Both 1 and 2
d) None
Solution: b)
Justification: It was one of the biggest cyberattacks that we have ever
seen impacting over 150 countries.
WannaCry is a type of malicious software (malware) classified as
ransomware. It encrypts essential files on your Windows device and
requires that you pay a ransom to unlock those files.
Note: Be careful what you click on. This malware was distributed by
phishing emails. You should only click on emails that you are sure came
from a trusted source.
Q Source:
http://indianexpress.com/article/opinion/columns/wannacry-
ransomware-attack-holding-to-ransom-4670661/

9. The Nobel Peace Prize laureate who is also the youngest United
Nations (UN) Messenger of Peace is
a) Antonio Guterres
b) Malala Yousafzai
c) Ban Ki-Moon
d) Kailash Satyarthi
Solution: b)
Learning: United Nations Messengers of Peace are distinguished
individuals, carefully selected from the fields of art, literature, science,
entertainment, sports or other fields of public life, who have agreed to
help focus worldwide attention on the work of the United Nations.
UN Secretary-General Antonio Guterres has appointed Nobel laureate
Malala Yousafzai (at 19, youngest such messenger) as a UN Messenger of
Peace to promote girls education.
It is the highest honour given by the United Nations. The messengers are
initially chosen for a period of three years.
They mainly promote the work of the UN agency they are ambassador
for.
Q Source: United Nations Website
10. The constitution grants both the President and the Governor
right to pardon punishments granted by the Judiciary. What is/are
the important differences between their powers?
1. President can only grant pardon on the advice of Union Council
of Ministers, whereas for the Governor this is purely a
discretionary matter.
2. President can grant pardon to a person awarded death sentence,
but Governor cannot do so.
3. President can exercise this power only in cases adjudicated by
the Supreme Court, whereas Governor can do this only in cases
under High Courts.
Select the correct answer using the codes below.
a) 1 only
b) 2 only
c) 1 and 3 only
d) 2 and 3 only
Solution: b)
Justification: Statement 1: The pardoning power of President is not
absolute. It is governed by the advice of the Council of Ministers. Same
applies for Governor as well.
Statement 2: There are only three major differences:
• President has the right to pardon punishments of sentences given
under Court Martial, whereas the governor does not have this
power.
• President can grant pardon to a person awarded death sentence.
But Governor of State does not enjoy this power.
• Governor’s pardoning power only extends to those areas where the
executive power of the concerned state extends. He cannot pardon
in case of offences committed under Central laws. So, 3 is
incorrect.
Q Source: Chapter on President: Indian Polity: M Laxmikanth
11. Consider the following with reference to the concept of time as
discussed in the Puranas.
1. The cycle of time is connected with the cyclical decline and
revival of dharma on earth.
2. One yuga follows the other, but the world once destroyed will
not be re-created.
Which of the above is/are correct?
a) 1 only
b) 2 only
c) Both 1 and 2
d) None
Solution: a)
Justification: The conception of time in the Puranas is mind-boggling.
There are four ages or yuhas — sat, treta, dwapara and kala, all
consisting of thousands and thousands of years.
These four yugas make up a mahayuga, and 1,000 mahayugas constitute
a kalpa.
Every kalpa is divided into 14 manvantaras, each presided over by a
Manu.
One yuga is followed by another, and the periodic destruction of the
world is followed by its re-creation. So, 2 is incorrect.
This cycle of time is connected with the decline and revival of dharma as
satyuga was considered to host the most moral society, and kalayuga the
least moral.
Q Source: Upinder Singh: A History of Ancient and Early Medieval
India
12. Ratooning is the practice of growing a Sugarcane crop
a) from the stubbles of previous crop
b) without the application of fertilizers or irrigation water
c) on hilly slopes with step cultivation
d) with symbiotic support from azolla and ferns
Solution: a)
Learning: When sugarcane is harvested, a portion of stalk is left
underground to give rise to a succeeding growth of cane, the ratoon or
stubble crop.
Ratooning utilizes this stubble, saves cost on preparatory tillage and
planting material. So, it gets benefit of residual manure and moisture.
Ratoon crop also matures earlier and gives more or less same yield as of
normal sugarcane.
However, only one ratoon should be taken because incidence of pests
and diseases increases and deterioration of soil takes place.
Q Source:
www.agriinfo.in/default.aspx?page=topic&superid=1&topicid=1089

13. Consider the following:


1. Rudralinga Bhairava, Kanha
2. Standing Buddha, Sarnath
3. Vishnu Anantasheshashayee, Deogarh
Which of the above is/are specimens of Gupta Sculpture?
a) 1 and 2 only
b) 2 and 3 only
c) 1 only
d) 2 only
Solution: b)
Learning: During the Gupta period the characteristic elements of the
Indian temple emerged and the plastic forms began to be used admirably
as an integral part of the general architectural scheme.
Statement 3: The stone carving from the temples at Deogarh and those
from the temples of Udayagiri and Ajanta are excellent specimens - four-
armed Vishnu is reclining gracefully on the coils of the Adisesha, whose
seven hoods form a canopy over his crowned head.
Statement 2: The magnificent red sandstone image of the Buddha from
Mathura is a most remarkable example of Gupta workmanship datable
to the 5th century A.D. The great Master, in all his sublimity, is here
shown standing with his right hand in abhayamudra
Q Source: http://ccrtindia.gov.in/guptasculp.php

14.With reference to “Government procurement preference to Make


In India order, 2017”, consider the following:
1. It gives preference to locally made goods and services as against
foreign goods in government procurements.
2. It restricts exports for India-made goods in order to satisfy
domestic demand thereby reducing unnecessary imports.
Which of the above is/are correct?
a) 1 only
b) 2 only
c) Both 1 and 2
d) None
Solution: a)
Justification: The national procurement policy will provide purchase
preference to local content in Government procurements.
Local content essentially means domestic value addition and local
suppliers are those whose goods or services meet minimum thresholds
(50%) for local content.
For the procurement of goods below Rs 50 lakhs, only local suppliers will
be eligible if the Nodal Ministry determines that there is a availability of
sufficient local capacity and local competition.
• The policy aims to maintain the balance between promoting ‘Make
in India’ and ensuring timely, quality and value-for-money
products for the procuring government entities.
• As the government procurement is always substantial it can
contribute towards increasing the participation of local employees
and improve their living conditions.
• Small procurements valued below Rs 5 lakhs are exempted from
the policy. The order also covers autonomous bodies and all the
government entities under the control of the government.
Q Source:
http://www.livemint.com/Industry/CY8Ex5fJ1OWuR8mkc7O84J/To-
Make-in-India-Modi-govt-approves-Buy-in-India.html

15. “Vande Mataram”, which eventually shaped our National Song, is a


poem composed by Bankim Chandra Chattopadhyay in 1870s,
which he included in his novel
a) Shaarda
b) Young Bengal
c) Anandamath
d) Geetanjali
Solution: c)
Learning: The first two verses of the song were adopted as the national
song of India in 1937.
• It played a vital role in the Indian independence movement, first
sung in a political context by Rabindranath Tagore at the 1896
session of the Indian National Congress.
• It became a popular marching song for political activism and
Indian freedom movement in 1905.
• Spiritual Indian nationalist and philosopher Sri Aurobindo
referred it as "National Anthem of Bengal".
Q Source: National Symbols of India

16.Seafaring fishermen in India are being issued tamper-proof and


machine-readable biometric identity cards. What will be the
benefit(s) of this move?
1. Infiltration of seaborne terrorists into mainland India will
become difficult.
2. Maritime security agencies will be able to monitor the real-time
location of fishing vessels with these fishermen onboard.
3. Satellite-linked emergency distress systems could be used to
save fishermen from natural hazards.
Select the correct answer using the codes below.
a) 1 only
b) 3 only
c) 1 and 2 only
d) 1, 2 and 3
Solution: d)
Justification: The government will place card readers (linked to a
centralised server through Internet) at harbours and authorised fish
landing centres for port authorities to record details of boats and
biometrically verify their declared crew before issuing them passes to
venture out to sea.
Statement 1: The process will be repeated when the fishermen make
landfall after their voyage.
To ensure compliance, the government will extend insurance for sea
accidents only to vessel owners, captains, and crew who follow the
procedure.
Statement 2: Maritime security agencies, police and port officials can
soon monitor the real-time location of fishing vessels on a digitised map
of the country’s seaboard.
They can pre-empt Indian fishermen from straying into foreign waters
(by sending them flash messages on mobile or wireless), coordinate sea
rescue missions better, and ensure that vessels return to harbours with
their crew intact and no outsiders or contraband on board.
Statement 3: Law enforcers patrolling the seas will be able to know the
details of a vessel at the wave of a handheld reader or by sending the
boat’s registration number as an SMS to a centralised server. This will
aid in disaster management.
• They can verify whether boat has been fined or impounded in the
past, its port of origin, the details of its owner and crew, the
amount of fuel and the nature of the fishing gear onboard etc.
• The system can also generate status reports to prove the
genuineness of Indian fishermen if they are detained by foreign
authorities. This would ensure their speedy release.
Q Source:
http://www.thehindu.com/news/cities/mumbai/news/Finally-
fishermen-get-biometric-cards/article14484056.ece

17. Stagflation occurs when these occur simultaneously:


a) High current account deficit and low inflation
b) Poor development prospects and high economic growth
c) Low money supply and booming investments
d) High inflation and low economic growth
Solution: d)
Justification: Stagflation refers to persistent high inflation coupled
with high unemployment and stagnant demand or growth in economy.
It raises a dilemma for economic policy, since actions designed to lower
inflation may exacerbate unemployment, and vice versa.
Learning: Economists offer two principal explanations for why
stagflation occurs.
First, stagflation can result when the productive capacity of an economy
is reduced by an unfavorable supply shock that causes an increase in the
price of oil for an oil-importing country.
• Such an unfavorable supply shock tends to raise prices at the same
time that it slows the economy by making production more costly
and less profitable
Second, both stagnation and inflation can result from inappropriate
macroeconomic policies. For example, central banks can cause inflation
by allowing excessive growth of the money supply, and the government
can cause stagnation by excessive regulation of goods markets and
labour markets.
Q Source: Economic Terms

18. Which of these is the largest glacier in India and second


longest glacier outside of the Polar Regions?
a) Zemu Glacier
b) Siachen Glacier
c) Chandra Glacier
d) Namik Glacier
Solution: b)
Justification: The world's largest glacier is the Lambert glacier in
Antarctica. Second largest is Siachen which is located in the eastern
Karakoram range in the Himalaya Mountains.
Gangotri Glacier is also one of the largest glaciers in the Himalayas.
Learning: The principal glaciers in Himalayas can be divided in four
groups:
• Punjab Himalaya Group of Glaciers
• Garhwal Himalaya Group of Glaciers
• Nepal Himalaya Group of Glaciers
• Assam Himalaya Group of Glaciers
Most glaciers lie in the states of Sikkim, Jammu and Kashmir, Himachal
Pradesh and Uttarakhand. Few glaciers are also found in Arunachal
Pradesh.
Q Source: Human Geography – The Land by Pradeep Sharma

19.Bacteria of the genus Rhizobium play a very important role in


agriculture by
a) Cutting down the salination and water logging in fields by
reverse osmosis
b) Bringing early ripening in fruits and vegetables
c) Inducing nitrogen-fixing nodules on the roots of legumes
d) Increasing the water content of the parent soil
Solution: c)
Learning: Fixation of nitrogen cannot be done independently by plants
from atmosphere. This must be fixed by an external agent.
Rhizobium are soil bacteria that fix nitrogen once they found a base
inside root nodules of legumes”.
Organisms that is responsible for nitrogen fixation belong to the group of
prokaryotes.
Q Source: 12th NCERT Biology

20. With reference to the history of India, consider the following


matches:
1. Akayanivi : A. Local well denoted only for royal use
2. Kupyadhyaksha : B. Superintendent of Forest Products
3. Kathasaritsagara : C. Collection of Indian legends
4. Office of Sthanika : D. Judge of Imperial court
Select the correct matches using the codes below.
a) 2 and 3 only
b) 1 and 4 only
c) 1, 2 and 3 only
d) 2 and 4 only
Solution: a)
Justification: Statement 1: Akayanivi denotes permanent endowment
of which interest was utilised for a specific purpose.
Statement 2: In the administration of Chandra Gupta Maurya, there was
a regular Forest Department administered by the Kupyadhyaksha
(Superintendent of Forest Products). His duty was to increase the
productivity of forests, to sell trees after fixing their price, classifying
strong trees etc.
Statement 3: It is a famous 11th-century collection of Indian legends,
fairy tales and folk tales as retold in Sanskrit by a Shaivite named
Somadeva.
Statement 4: There was an intermediate level of administration between
the district level and that of the village. The unit here was formed by a
group of five or ten villages. The two important officials concerned with
the administration of this unit were the gopa and the sthanika.
The tax was collected by the sthanika who worked directly under the
Pradesika.
Q Source: Improvisation: Past year UPSC papers

21. The organizations that oversee the keeping of international


standards of measurement are
1. International Bureau of Weights and Measures (IBWM)
2. United Nations Trusteeship Council (UNTC)
3. General Conference on Weights and Measures
4. International Union for Conservation of Nature (IUCN)
Select the correct answer using the codes below.
a) 1 and 3 only
b) 2, 3 and 4 only
c) 1, 2 and 3 only
d) 1 and 4 only
Solution: a)
*Justification*: UNTC and IUCN can be eliminated. The only answer
left is A.
The Metre Convention of 1875 covered standards for the metre and the
kilogram, but later expanded to include other metrics as well.
The treaty established three international organisations to oversee the
keeping of international standards of measurement: General Conference
on Weights and Measures; International Committee for Weights and
Measures (ICWM) and International Bureau of Weights and Measures
(IBWM).
The International System of Quantities (ISQ) is a system based on seven
base quantities: length, mass, time, electric current, thermodynamic
temperature, amount of substance, and luminous intensity.
Q Source: Improvisation: 9th NCERT Science

22. Independent of Judiciary is helpful in a Federal System


because
1. It helps prevent usurpation of authority by higher levels of
government.
2. It allows impartial dispute resolution between different tiers of
the government.
Which of the above is/are correct?
a) 1 only
b) 2 only
c) Both 1 and 2
d) None
Solution: c)
Justification: Statement 1: Many of the Supreme Court judgments
have helped the states balance the authority of the Centre. This can be
best seen in the case of inter-state water disputes where states have
taken disputes to the Supreme court despite an award from a water
tribunal.
Statement 2: Independence is crucial to unbiased dispute resolution.
If the Supreme court was regulated by the Union government, we could
not expect it to take sides with the states on crucial issues such as inter-
state water disputes as mentioned above.
Q Source: General questions on Polity

23. The “Coral Triangle” region is home to the highest coral


diversity in the world and there is broad scientific consensus that it
represents a global epicenter of marine life abundance and
diversity. This triangle is located in
a) Bermuda in Pacific Ocean
b) South-east Asia
c) North Atlantic shores of Europe
d) Berring Strait near Arctic Circle
Solution: b)
Importance: Spanning only 1.6% of the planet’s oceans, the Coral
Triangle region is home to 76% of the world’s known coral species.
• It contains the highest reef fish diversity with 37% of the world’s
reef fish species concentrated in the area.
• It is also a spawning and nursery ground for six species of
threatened marine turtles, endangered fish and cetaceans such as
tuna and blue whales.
Learning: Recognizing the need to safeguard the region’s marine and
coastal resources, the Coral Triangle Initiative on Coral Reefs, Fisheries
and Food Security (CTI-CFF) was launched in 2009.
The CTI-CFF is a multilateral partnership between the governments of
Indonesia, Malaysia, Papua New Guinea, Philippines, Solomon Islands
and Timor-Leste (the ‘CT6’).
Q Source: http://www.coraltriangleinitiative.org/

24. The Rowlatt committee was appointed by the British Indian


Government to
a) Suggest a new structure for responsible government in India
b) Create a definitive tax arrangement between princely states and
British provinces
c) Evaluate political terrorism in India and suggest measures to
curb the same
d) Examine educational schemes of the Indian Universities
Solution: c)
*Learning*: It was instituted towards the end of World War I when the
Indian Revolutionary movement had been especially active and had
achieved considerable success.
• The purpose was to evaluate political terrorism in India, especially
Bengal and Punjab, its impact, and the links with the German
government and the Bolsheviks in Russia.
• These movements even received support from Germany which
planned to destabilise British India.
• The Rowlatt committee recommended extending the Defence of
India Act (as the Rowlatt act) to thwart any possible revival of the
organizations like Samiti in Bengal and the Ghadarite movement in
Punjab.
• The Defence of India Act 1915 was an emergency criminal law with
the intention of curtailing the nationalist and revolutionary
activities during and in the aftermath of the First World War.
Q Source: Spectrum Modern India by Rajiv Ahir

25. Who among the following is widely regarded as having


inspired the Pakistan Movement and called the "Spiritual father of
Pakistan"?
a) Maulana Abdul Kalam Azad
b) Khan Abdul Gaffar Khan
c) Muhammad Iqbal
d) Syed Ahmed Khan
Solution: c)
Learning: Iqbal became a member of the London branch of the All-
India Muslim League.
• Later, during the League's December 1930 session, he delivered his
most famous presidential speech known as the Allahabad Address
in which he pushed for the creation of a Muslim state in Northwest
India.
• Widely known as Allama Iqbal, he is considered one of the most
important figures in Urdu literature.
• However, it was Choudhary Rahmat Ali who used the word
Pakstan (without the letter “I”) for the first time and was presented
in the Round Table conferences in 1933.
Q Source: Spectrum Modern India by Rajiv Ahir

26. "SANKALP" scheme of the Government of India is related to


a) Open Massive Online Coursewares for school students
b) Construction works to make public places amenable to
differently abled and transgender population
c) Skill development and augmentation of knowledge to promote
employment
d) Eradicating caste-based harassment from Indian soil
Solution: c)
Learning: During 2017-18, the Government will launch the Skill
Acquisition and Knowledge Awareness for Livelihood Promotion
programme (SANKALP) at a cost of four thousand crore rupees.
• SANKALP will provide market relevant training to over three crore
youth, which has been felt necessary in the fast-changing economic
scenario.
• The next phase of Skill Strengthening for Industrial Value
Enhancement (STRIVE) will also be launched in 2017-18 at a cost
of over rupees two thousand crores.
• STRIVE will focus on improving the quality and market relevance
of vocational training provided in ITIs and strengthen the
apprenticeship programmes through industry cluster approach.
Q Source: Yojana March 2017

27. LCA Tejas, recently in news, is a/an indigenously designed


a) Combat aircraft
b) Electric motor vehicle
c) High Performance Diesel
d) Solar Cell
Solution: a)
Learning: It is a Light Combat Aircraft (LCA).
• It is set to replace the MiG-21 series. It has better avionics and
improved cockpit. Since Tejas uses carbon composites in its
structure, it is lighter in weight and has a much stronger body
• It also has features of stealth fighter jet.
• Tejas will have a limited reach of a little over 400-km, which
means it can be used for close air-to-ground operations.
• For any strikes which happen deep into enemy territory will have
to be undertaken by Russian-origin Sukhoi-30MKIs or the Rafales.
Q Source: http://economictimes.indiatimes.com/news/defence/first-
squadron-of-lca-tejas-inducted-into-indian-air-force-8-things-to-
know/articleshow/53001820.cms

28. The United States of America recently withdrew from which


of these major trade blocs via an executive order?
a) Trans-Pacific Partnership (TPP)
b) North American Free Trade Bloc (NAFTA)
c) Eurozone
d) Trans-Atlantic Partnership
Solution: a)
Learning: TPP is a trade agreement between Australia, Brunei, Canada,
Chile, Japan, Malaysia, Mexico, New Zealand, Peru, Singapore, the
United States (before the executive order) and Vietnam.
Trump had blamed TPP for job losses and growing discontent among
businessmen in America.
The move also intensified speculation over the future of the 23-year-old
North American Free Trade Agreement (NAFTA).
Q Source: https://www.theguardian.com/us-
news/2017/jan/23/donald-trump-first-orders-trans-pacific-partnership-
tpp

29. India has imposed several restrictions on the political


activities of serving civil servants. Which of these are among those
restrictions?
1. She cannot become a member of a political party.
2. She cannot voice her opinion on any matter on a social media
platform.
3. She doesn’t enjoy a right to vote.
4. She is prohibited from participating in a strike.
Select the correct answer using the codes below.
a) 1 and 2 only
b) 2 and 4 only
c) 1 and 4 only
d) 2 and 3 only
Solution: c)
Justification: Political neutrality of civil servants has been regarded as
one of cardinal conditions for the success of a democratic government.
Statement 1: If they are engaged in political activities, it violates the
above principle. They cannot contest Lok Sabha or State assembly
elections.
Statement 2: Civil servants are not allowed to criticise govt on social
media or press. These do not apply for general
Statement 3: They do enjoy a right to vote as is the right of every voter in
India.
Statement 4: Striking by civil servants constitutes a disciplinary offence.
Q Source: Governance In India By Laxmikanth

30. With reference to a region, consider the following:


1. World's first fluorescent frog was discovered here recently.
2. A transparent frog that carries its heart on skin was recently
discovered here.
The region is?
a) Andaman and Nicobar Islands
b) South America
c) Arctic Circle
d) Siberian Hinterlands
Solution: b)
*Learning*: Options were such that you could eliminate easily.
Statement 1: In normal light the frog appears to have a dull, mottled
browny-green skin with red dots, but under UV light it glows a bright
fluorescent green.
Statement 2: Earlier, In the Neotropics, there was a group of so-called
glassfrogs that amaze with their transparent skin covering their bellies
and showing their organs underneath.
However, a recently discovered new species from Amazonian Ecuador,
goes a step further to fully expose its heart thanks to the transparent skin
stretching all over its chest as well as tummy.
Q Source:
https://www.sciencedaily.com/releases/2017/05/170529085609.htm
https://www.theguardian.com/world/2017/mar/14/worlds-first-
fluorescent-frog-discovered-in-south-america

31. With reference to Medieval India, what did Khutbah refer to?
a) A sermon read in the name of the ruling king on the occasion of
Friday prayers
b) A group of merchants who owned large caravans and
transported goods from market to market
c) Muslim devotee who emphasized the personal devotion of man
to God through love
d) A docket that contained the core instructions of the ruler
Solution: a)
Learning: Option B were Banjaras (this was the place from where
UPSC probably lifted the Banjara question in 2016 – some of these
keywords were mentioned with definitions at the Q Source).
Option C is sufis.
• Khutabh serves as the primary formal occasion for public
preaching in the Islamic tradition.
• Such sermons occur regularly, as prescribed by the teachings of all
legal schools.
• The Islamic tradition can be formally at the dhuhr (noon)
congregation prayer on Friday.
Q Source: Medieval India – Romila Thapar: Class VII: Old NCERT
32. The ‘Man Ki Baat’ programme on All-India Radio is hosted
by
a) President of India
b) Prime Minister
c) Member of Parliaments in rotation
d) Major celebrities working in development sector
Solution: b)
Learning: In the programme, he addresses the people of the nation on
radio, DD National and DD News.
Each month, some selected calls become a part of the broadcast.
The President of the United States was a part of one January edition of
'Mann Ki Baat', aired in 2015.
Q Source: Yojana – May 2017

33. Consider the following statements:


1. Rainfall consistently decreases with gain in altitude in North-
eastern India.
2. Rainfall increases from west to east in the Northern Plains.
Which of the above is/are correct?
a) 1 only
b) 2 only
c) Both 1 and 2
d) None
Solution: b)
Justification: Statement 1: In the North-east, the hilly terrains and
mountainous environment exert a great control on the generation and
distribution of precipitation because orography provides the necessary
uplift to the moisture-laden currents striking against the topographic
barriers.
As a general rule, precipitation increases with elevation to a certain level
of altitude and then decreases after a certain level with further ascent up
the slopes.
Statement 2: It increases from east to west or say decreases from west to
east.
As we move from Meghalaya to Haryana or Punjab in Northern plains,
we observe that the rainfall decreases.
In peninsular India, rainfall decreases from coast to interior parts.
Q Source: Human Geography: The Land – By Pradeep Sharma

34. If you stay long in a busy traffic area, you often feel
lightheadedness, confusion, headache due to the haemoglobin in
your blood combining with X. What is X here?
a) Carbon dioxide
b) Carbon monoxide
c) Methane
d) Sulphur dioxide
Solution: b)
Learning: Carbon monoxide poisoning occurs after breathing in too
much carbon monoxide (CO). The above are symptoms of mild acute
poisoning.
Carbon monoxide causes adverse effects by combining with hemoglobin
to form carboxyhemoglobin (HbCO) in the blood. This prevents
hemoglobin from carrying oxygen to the tissues.
Carboxyhemoglobin can revert to hemoglobin, but the recovery takes
time
Q Source: 8th Science NCERT
35. Government of India implements the Integrated Child
Development Scheme (ICDS) via
a) Anganwadi centres
b) Krishi Kiosks
c) Common Service Centres (CSCs)
d) Single Window Facilitation Centres (SWFCs)
Solution: a)
*Learning*: It provides food, preschool education, and primary
healthcare to children under 6 years of age and their mothers. These
services are provided from Anganwadi centres established mainly in
rural areas and staffed with frontline workers.
In addition to fighting malnutrition and ill health, the programme is also
intended to combat gender inequality by providing girls the same
resources as boys.
Target group is children in the age of 0-6 years, pregnant women and
lactating mothers.
Q Source: Important Government Schemes

36. What is/are the main difference(s) between ‘Limited Legal


Tender Money’ and ‘Unlimited Legal Tender Money’?
1. The former can be issued only by the Reserve Bank of India,
whereas the latter can be issued only by the Government.
2. The former are mainly cheques and bank drafts which cannot be
used to settle retail payments, whereas the latter can be used to
settle retail payments.
Which of the above is/are correct?
a) 1 only
b) 2 only
c) Both 1 and 2
d) None
Solution: d)
*Justification*: The conceptual explanation will clear both statements.
The legal tender money is of two types:
Limited Legal Tender Money: This is a form of money, which can be paid
in discharge of a debt up to a certain limit, and beyond this limit a
person may refuse to accept the payment and no legal action can be
taken against. Coins are limited legal tender in India. So, 1 is wrong, as
government issues coins, whereas RBI issues notes.
Unlimited Legal Tender Money: A person who refuses to accept this
money, a legal action can be taken against. Paper notes / currency are
unlimited legal tender in India. So, 2 is wrong.
There is also a Non-Legal Tender Money: It is a form of money which is
generally accepted, but legally it is not bound to accept it. Cheques, bank
drafts, bills of exchange, postal orders etc are not legal tenders and are
accepted only at the option of the creditor, lender or seller.
Q Source: Yojana: February 2017

37. Prime Minister’s 10-point Agenda on Disaster Risk


Reduction Outlined at the Asian Ministerial Conference on
Disaster Risk Reduction (AMCDRR) 2016 include
1. Development sectors must imbibe the principles of disaster risk
management
2. Investment in risk mapping
3. Leverage technology to enhance the efficiency of our disaster
risk management efforts
4. Bring about greater cohesion in international response to
disasters
Select the correct answer using the codes below.
a) 2 and 4 only
b) 1 and 3 only
c) 4 only
d) 1, 2, 3 and 4
Solution: d)
*Justification*: Statements and options in these questions are set in a
way as to make answering easy. UPSC does the same. Take full
advantage of this in unknown questions.
This was the first Asian Ministerial Conference for Disaster Risk
Reduction after the advent of the Sendai Framework was hosted by the
Government of India in November 2016.
The Agenda also proposes, inter alia:
• Work towards risk coverage for all-starting from poor households
to SMEs to multinational corporations to nation states.
• Encourage greater involvement and leadership of women in
disaster risk management.
• Develop a network of universities to work on disaster issues.
• Utilize the opportunities provided by social media and mobile
technologies.
Q Source: Yojana: January 2017

38. One of the most important buildings in Fatehpur Sikri, it is


also known as the “Dream of Stones”:
a) Panch Mahal
b) Jantar Mantar
c) Meenakshi Temple
d) Hawa Mahal
Solution: a)
*Justification*: All options could be eliminated except A.
Hawa Mahal is a palace in Jaipur, so eliminated. Jantar Mantar (there
are many, but the famous one) is in Delhi. Meenakshi Temple is in TN.
Learning: The Panch Mahal is known as the wind catcher tower. It was
commissioned by sikarwar rajputs.
This is an extraordinary structure employing the design elements of a
Buddhist Temple; entirely columnar, consisting of four stories of
decreasing size arranged asymmetrically upon the ground floor.

Q Source: Improvisation: Past year SSC papers

39. With reference to the “BioTrade Initiative”, consider the


following:
1. It deals only with products made using exotic or invasive
species.
2. It was established as a binding protocol under the International
Union for Conservation of Nature (IUCN).
Which of the above is/are correct?
a) 1 only
b) 2 only
c) Both 1 and 2
d) None
Solution: d)
Background: Many developing countries which are endowed with rich
biodiversity resources are losing it at an alarming rate despite the
dependence of many communities on such resources for livelihood.
• However, sustainable use of biological resources can generate
tangible economic benefits for such populations, if they could
themselves support development and nature conservation.
• One of the ways to achieve this is to take advantage of the new
investment and trade opportunities that are emerging for
biodiversity-based products and services.
Concept: BioTrade refers to those activities of collection, production,
transformation, and commercialization of goods and services derived
from native biodiversity under the criteria of environmental, social and
economic sustainability.
Justification: Statement 1 and 2: The BIOTRADE Initiative was
launched in 1996 by UNCTAD with the objective of stimulating trade and
investment in biological resources to further sustainable development, in
line with the objectives of the Convention on Biological Diversity.
BIOTRADE seeks to enhance the capability of developing countries to
produce value-added products and services from biodiversity for both
domestic and international markets.
Q Source: http://unctad.org/en/Pages/DITC/Trade-and-
Environment/BioTrade.aspx

40. The Ministry of Social Justice and Empowerment had


launched a dedicated Venture Capital Funds Scheme for which of
these communities?
a) Scheduled Castes
b) Tribals of North-eastern states
c) Widows who are part of Self-Help Groups
d) Rural school children
Solution: a)
Learning: The objective of the Venture Capital Fund Scheme for
Scheduled Caste Entrepreneurs is to promote entrepreneurship among
India’s scheduled caste population.
• The eligibility criteria for selecting the SC entrepreneurs is as
follows:
• The projects/units being set up in manufacturing and services
sector ensuring asset creation out of the funds deployed in the unit
shall be considered;
• While selecting the SC entrepreneurs, women SC entrepreneurs
would be preferred
• Companies having at least 60% stake holdings by Scheduled Castes
entrepreneurs for the past 12 months.
• The companies applying for assistance of more than Rs. Five crore,
shall preferably get their project appraised by the banks / FIs
before approaching for assistance under the scheme.
Q Source: http://socialjustice.nic.in/SchemeList/Send/38?mid=24541

41.The Micro Solar Dome (MSD) is a clear and green energy initiative
of the Department of Science and Technology. Consider the
following with reference to it.
1. It is a low cost solution to the industries for off-grid mass power
generation using captive plants at remote locations.
2. It is designed to be installed by restaurants and large kitchens
with large open spaces to replace gas cooking with solar-
powered cooking.
Which of the above is/are correct?
a) 1 only
b) 2 only
c) Both 1 and 2
d) None
Solution: d)
Justification: None of the above is correct.
• It is a tiny device that basically captures day light and provides the
same inside a dark room (typically in a slum that are closed from
all sides even during the day without windows).
• So, it is a day and night lighting device without needing any bulbs.
No opening on the roof is required to let the sunlight come in.
• A transparent semi-spherical upper dome made of acrylic material
captures the sunlight and the light passes through a sun-tube
having a thin layer of highly reflective coating on the inner wall of
the passage. This allows for a bright exposition.
• It is leak proof and works throughout the day and 4 hours
continuously after sunset.
• The potential users of this device are the 10 million off-grid
households in urban and rural spaces.

Q Source: http://pib.nic.in/newsite/PrintRelease.aspx?relid=153493

42. `Swadeshi’ and ‘Boycott’ were adopted as methods of


struggle for which of these causes in 1905?
a) Establishment of Hindu Mahasabha
b) Agitation against the Partition of Bengal
c) Visit of Cabinet Mission
d) Opposition to formation of Indian National Congress (INC)
Solution: b)
Learning: These methods were used for the first time in the history of
Indian freedom movement.
This started the organized phase of freedom struggle and attracted the
intellectuals.
Later Gandhiji entered the scenes and gave a new direction to swadeshi
and boycott starting the mill strikes, Champaran and later non-
cooperation movement.
Q Source: Basics of Indian freedom Movement

43. In the Parliament, the main object of Adjournment motion is


to
a) Draw the attention of the House to a recent matter of urgent
public importance
b) Prorogue a session of the House
c) Prevent the passing of a public bill
d) Invite the President to a meeting of the House
Solution: a)
Learning: It is introduced in the Parliament to draw attention of the
House to a definite matter of urgent public importance, and needs the
support of 50 members to be admitted.
• As it interrupts the normal business of the House, it is regarded as
an extraordinary device.
• It involves an element of censure against the government and
hence Rajya Sabha is not permitted to make use of this device.
• The discussion on an adjournment motion should last for not less
than two hours and thirty minutes.
Q Source: Chapter on Parliament: Indian Polity: Laxmikanth

44. Absentee landlordism in colonial India is often associated


with which of these British measures?
a) Permanent Settlement System 1793
b) Pitts India Act 1784
c) Consent Act 1891
d) Montagu–Chelmsford Reforms
Solution: a)
Learning: In Bengal while the zamindars were landlord over the
tenants, the Permanent settlement system reduced himself to a
subordinate of the East India company.
• The Zamindar was to make a fixed annual payment to the
government, retaining one tenth of his collection as his fee.
• Since at first the rates were high and as prices did not rise, he could
not extract high payments from the peasant.
• Unable to pay, many zamindars fled. Moreover, a lot of them were
better off living luxurious life in the urban side rather than
cultivating their large plots.
• So, the result was a big change in Zamindari personnel and the
appearance of new men from Calcutta who bought estates as
financial speculations.
• The new landlords were often absentees with no local connections.
Q Source: 8th NCERT: Our Pasts – Part I

45. Consider the following about some un-conventional gas


sources.
1. Tight gas is natural gas with highest density that instantly leaks
from reservoir rocks on first impact at high pressure.
2. Coal Mine Methane (CMM) refers to methane released from the
coal and surrounding rock strata due to mining activities.
Which of the above is/are correct?
a) 1 only
b) 2 only
c) Both 1 and 2
d) None
Solution: b)
Justification: Statement 1: Tight gas is natural gas produced from
reservoir rocks with such low permeability that massive hydraulic
fracturing is necessary to produce the well at economic rates.
• A tight gas reservoir is one that cannot be produced at economic
flow rates or recover economic volumes of gas unless the well is
stimulated by a large hydraulic fracture treatment and/or
produced using horizontal wellbores.
• This definition also applies to coalbed methane, shale gas, and
tight carbonate reservoirs.
Statement 2: There is a difference between coalbed methane (CBM) and
coal mine methane (CMM).
• CBM refers to methane that is found in coal seams. It is formed
during the process of coalification, the transformation of plant
material into coal.
• CMM refers to methane released from the coal and surrounding
rock strata due to mining activities.
• Like CBM, CMM is a subset of the methane found in coal seams,
but it refers specifically to the methane found within mining areas
(e.g., within a mining plan), while CBM refers to methane in coal
seams that will never be mined.
Learning: In underground mines CMM can create an explosive hazard
to coal miners, so it is removed through ventilation systems. In
abandoned mines and surface mines, methane might also escape to the
atmosphere through natural fissures or other diffuse sources.
Q Source: Improvisation: August 2016 : Yojana

46. In the context of sustainable development, the Indian State


that has become the first carbon-neutral State is
a) Chhattisgarh
b) Himachal Pradesh
c) Maharashtra
d) Punjab
Solution: b)
Learning: Himachal Pradesh had mandated all government
departments to begin environment audit. It also has a carbon-neutral
policy.
It was also the first state in the country to have a State Data Centre
(SDC), designed using green data centre concept.
The people of Himachal Pradesh will be expected to undergo lifestyle
changes, according to the environment plan and ‘Policy and Strategy on
Climate Change and Harnessing Carbon Credit.
The focus is on carrying developmental works without causing harm to
flora and fauna.
Q Source: http://economictimes.indiatimes.com/et-cetera/himachal-
set-to-become-indias-1st-carbon-free-state/articleshow/4000331.cms
http://www.dnaindia.com/india/report-majuli-all-set-to-become-india-
s-1st-carbon-neutral-district-sonowal-2363044

47. The Government has launched PowerTex India for


a) Mandating organic cotton production for use in all textile mills
b) Increasing the tax base in the textile sector
c) Boost common infrastructure and modernisation of the
powerloom sector in the country
d) Both (a) and (b)
Solution: c)
*Learning*: It was a comprehensive scheme for powerloom sector
development.
It has overall nine major components, including two new schemes:
• Pradhan Mantri Credit Scheme (PMCS) for powerloom weavers:
where Financial assistance, including subsidy and interest
reimbursement, will be given as against the credit facility under
Pradhan Mantri Mudra Yojana to the decentralised power loom
units.
• Solar energy scheme (SEC) for powerlooms: where Financial
subsidy for the installation of the Solar Photo Voltaic Plants will be
provided to alleviate the problems of power cuts.
Q Source: http://pib.nic.in/newsite/PrintRelease.aspx?relid=160404

48. ‘Agriculture Marketing and Farmer Friendly Reforms Index’


to sensitise India’s states about the need to undertake reforms has
been developed by
a) Food and Agriculture Organization (FAO)
b) Indian Council of Agricultural Research (ICAR)
c) NITI Aayog
d) World food Programme and Oxfam
Solution: c)
Learning: It will sensitise states about the need to undertake reforms in
the three key areas of Agriculture Market Reforms, Land Lease Reforms
and Forestry on Private Land (Felling and Transit of Trees).
• The index carries a score with a minimum value “0” implying no
reforms and maximum value “100” implying complete reforms in
the selected areas.
• As per NITI Aayog’s index, Maharashtra ranks highest in
implementation of various agricultural reforms.
• Gujarat ranks second closely followed by Rajasthan and Madhya
Pradesh.
• Almost two third States have not been able to reach even the
halfway mark of reforms score, in the year 2016-17.
• The index aims to induce a healthy competition between States and
percolate best practices in implementing farmer-friendly reforms.
Q Source: NITI Aayog Website

49. Consider the following with reference to malnutrition in


India.
1. It constitutes both under-nutrition and over-nutrition.
2. India has the poorest rank among South Asian nations in Global
Hunger Index (GHI).
3. Under-nutrition is more prevalent in rural areas than urban
areas.
4. Poor sanitation is considered as one of the major reasons of
malnutrition.
Select the correct answer using the codes below.
a) 3 and 4 only
b) 1, 3 and 4 only
c) 1 and 2 only
d) 1, 2, 3 and 4
Solution: b)
Justification: Statement 2: The 2015 Global Hunger Index (GHI)
Report ranked India 20th amongst leading countries with a serious
hunger situation. Amongst South Asian nations, it ranks third behind
only Afghanistan and Pakistan with a GHI score of 29.0 ("serious
situation").
Statement 3: This is mainly due to low socio-economic status. Anaemia
for both men and women is only slightly higher in rural areas than in
urban areas. In urban areas, overweight status and obesity are higher
than rural areas.
Statement 4: UNICEF has recognized that poor sanitation in India is one
of the important reasons for malnutrition. This occurs by ingestion of
certain bacteria in intestine that prevent proper digestion of food and
lead to diseases.
Q Source: Important Socio-economic issues

50. With reference to pre-independent India, Hindustan


Socialist Republican Association (HSRA) was a
a) British think tank to neutralize Indian political groups
b) Nationalist revolutionary organisation
c) Consortium of moderates exiled from Congress
d) A publication that vouched equal rights for native Indians and
British rulers
Solution: b)
*Learning*: Initially known as Hindustan Socialist Republican Army, it
was established in 1928 in New Delhi by Chandrasekhar Azad, Bhagat
Singh, Sukhdev Thapar and others.
• It was founded due to the discontent that arose as a result of
suspension of non-cooperation movement and the apparently poor
policies of the moderates.
• The policies of Gandhi were criticised and youths were called to
join the organisation.
• Later it turned into HSRA.
• The HSRA's manifesto titled Philosophy of the Bomb was written
by Bhagawati Charan Vohra. Sanyal wrote a manifesto for the HRA
entitled Revolutionary.
• One of the major actions of HSRA was the bombing of the Central
Legislative Assembly in Delhi in 1929 for which Bhagat Singh and
others were sentenced to death.
Q Source: Spectrum Modern India by Rajiv Ahir
51. With reference to the ‘Jal Marg Vikas’ project financially supported
by the World Bank, consider the following:
1. It is being developed on Peninsular Rivers.
2. It will help develop navigation facilities for commercial water
vessels.
3. It will connect all urban drinking water supply connections to a
single grid operable from a central location.
Select the correct answer using the codes below.
a) 2 only
b) 1 only
c) 1 and 3 only
d) 2 and 3 only
Solution: a)
*Justification*: Statement 1: Peninsular rivers are fast flowing and not
very navigable. So, you could eliminate statement 1.
Statement 3: Doing this is a mammoth and unnecessary task. Statement
3 could also have been eliminated.
Statement 2: It is a project on the river Ganga , being developed
between Allahabad and Haldia to cover a distance of around 1620 kms
on National Waterway-1 (NW-1).
• The project envisages development of a fairway with three metres
depth , which would enable commercial navigation of at least 1500
ton vessels on the river.
• Construction of multi modal terminals, jetties, river information
system, channel marking, navigational lock, river training and
conservancy works are to be undertaken as part of the project.
Q Source:
http://www.thehindubusinessline.com/economy/logistics/jal-marg-
vikas/article9711712.ece
52. It has been observed that agricultural prosperity does not
depend as much as on the total net sown area as it does on the
percentage of net sown area to the total reporting area. Which of
these states has the maximum percentage of net sown area to total
area?
a) Assam
b) Rajasthan
c) Punjab
d) Kerala
Solution: c)
Learning: Punjab (83.7%), Haryana (80.3%), Uttar Pradesh (62.8%),
West Bengal (60.3%), Maharashtra (58.3%) and Kerala (57.6%) have the
highest net sown areas in percentage terms.
• Madhya Pradesh has the largest net sown area (tota) which is
about 14% of the total reporting net sown area of India.
• Most parts of the peninsular plateau, Assam and Rajasthan have
medium proportion of 25 to 50% of the net sown area to the total
area.
• Most of the hilly areas in the states of Jammu and Kashmir,
Himachal Pradesh and the north-eastern states have low
proportion of less than 25 per cent.
Q Source: Human Geography: The Economy – by Pradeep Sharma

53. Unemployment rate usually temporarily rises when an


economy begins to recover from a recession. This may be because
a) Recovery from recession leads to drastic fall in demand of goods
and services
b) Labour force participation rate increases as more workers are
encouraged to find jobs in the hope of recovery
c) Large highly mechanized firms do not recover, as quickly as
small firms, which tends to depress employment
d) Data collection is suspended in most nations for a long period of
time after recession
Solution: b)
Learning: The unemployment rate is calculated by dividing the number
of unemployed individuals by all individuals currently in the labour force
(i.e. working or looking for work).
During recession, a lot of workers stop finding jobs, which reduces the
Labour force participation rate (LFPR).
As the economy begins to recover from a recession, workers who were
previously discouraged about their chances of finding a job begin to look
for work again.
This raise the LFPR temporarily, even while the total number of
employed people remains the same. This naturally leads to a temporary
increase in unemployment rate despite the economic recovery.
Q Source: Improvisation: Indian Economy: Ramesh Singh

54. In India, the Inner Line Permit (ILP) system is operational in


a) All states with International borders
b) All designated Scheduled Areas under the Constitution
c) All North-Eastern States
d) None of the above
Solution: d)
Justification: This permit is presently operational in only three north-
eastern states viz. Arunachal Pradesh, Nagaland and Mizoram. So, D is
the answer.
Manipur was earlier regulated by Inner Line permit system (ILP), which
was later abolished.
The ILP is a special pass or permit or quasi visa that is required by
Indian citizens to enter protected/restricted area of state for a limited
period.
It was introduced by then British India government to protect
indigenous tribal people from encroachment into their areas by
outsiders.
Later, they used it as an instrument to advance the commercial interest.
Q Source: Insights Polity Module 2017

55. With reference to the “Middle Income Group Scheme”,


consider the following:
1. Ministry of Law and Justice had introduced this scheme.
2. Exclusive posts in the government and judiciary will be reserved
to selected castes from middle income groups under the scheme.
3. The Chief Justice of India will be the Patron-in-Chief of the
society created under this scheme.
Select the correct answer using the codes below.
a) 1 and 3 only
b) 2 only
c) 2 and 3 only
d) 3 only
Solution: d)
Justification: Statement 1: The Supreme Court has introduced Middle
Income Group Scheme for providing legal services to the middle and
relatively lower income groups. So, 1 and 2 are wrong.
• Under the scheme, middle class people who can`t afford the
expensive litigation in the Supreme Court can avail the services of
the society for a nominal amount.
• This is for citizens whose gross income is not exceeding Rs.60, 000
per month or Rs. 7,50,000 per annum.
Statement 2: A case will be registered under the MIG Legal Aid Scheme
and forwarded to Advocate-on-Record/ Senior Counsel/Arguing Counsel
on the panel for their opinion.
Statement 3: A society will be created. The Chief Justice of India is the
Patron-in-Chief of the society with Attorney General its ex-officio Vice
President, Solicitor General its Honorary Secretary and other senior
advocates of the apex court as its members.
Q Source: http://pib.nic.in/newsite/PrintRelease.aspx?relid=158509

56. The government has introduced Inter-State River Water


Disputes (Amendment) Bill, 2017 in Lok Sabha in order to
a) Delegate jurisdiction of inter-state water disputes to Supreme
Court
b) Move inter-state water disputes into the State List
c) Set up a single permanent Tribunal to adjudicate all inter-State
river water disputes
d) Invalidate all judicial pronouncements with respect to river
disputes
Solution: c)
Learning: This body will subsume existing tribunals.
• The permanent tribunal will have retired Supreme Court judge as
its chairperson.
• The amendment also proposes to set up Dispute Resolution
Committee (DRC).
• The DRC, comprising experts and policy-makers, is proposed to
handle disputes prior to the Tribunal.
• The government has also proposed to float some Benches by
amending the Inter-State Water Disputes Act, 1956.
• The benches will look into disputes as and when required and will
cease to exist once the disputes are resolved.
• The total time period for adjudication of dispute has been fixed at
maximum of four and half years.
• As per the present version of the bill, the decision of the Tribunal
shall be final and binding with no requirement of publication in the
official Gazette.
Q Source: http://pib.nic.in/newsite/mbErel.aspx?relid=159201
57. GBU-43/B, recently seen in news, is the
a) Solar powered drone owned by Inter-Services Intelligence (ISI)
b) World’s largest non-nuclear weapon
c) Stock of a malware company
d) First biological battery brought under commercial application
Solution: b)
Learning: It is dubbed as the ‘Mother of all bombs’. It has been
dropped recently on an Islamic State complex in Afghanistan by the US
forces.
• It is also called as the Massive Ordnance Air Blast (Moab).
• It is designed to destroy heavily reinforced targets or to shatter
ground forces and armour across a large area.
• Its blast is equivalent to 11 tons of TNT. By comparison, the nuclear
weapon dropped on Hiroshima had a blast yield of 15 tons of TNT.
• The bomb leaves no lasting radiation effect because it is non-
nuclear.
Q Source: Insights PT Full Module 2017

58. CoBRA (Commando Battalion for Resolute Action) is a


specialised unit of the
a) Central Reserve Police Force (CRPF)
b) Indian Coast Guard
c) Border Security Force (BSF)
d) Central Industrial Security Force (CISF)
Solution: a)
Learning: CoBRA of India is proficient in guerrilla tactics and jungle
warfare.
Originally established to counter the Naxalite problem, CoBRA is
deployed to address any insurgent group engaging in asymmetrical
warfare.
Currently numbering ten battalions, CoBRA is ranked among one of
India’s more experienced and successful law enforcement units under
the CRPF.
Q Source: Indian Security Forces

59. The Composite Water Management Index recently developed


by NITI Aayog will cover which of these key performance
indicators to aid in efficient management of water resources?
1. Creation of irrigation potential
2. Restoration of Water bodies
3. Assets created under watershed development programme
4. On-farm power management
Select the correct answer using the codes below.
a) 2 only
b) 3 and 4 only
c) 1 and 4 only
d) 1, 2, 3 and 4
Solution: d)
Justification: The Index has 28 key Performance Indicators (KPIs)
covering irrigation status, drinking water and other water-related
sectors.
You can find the entire list here
http://niti.gov.in/writereaddata/files/document_publication/WMI-
Document.pdf
Major categories are:
• Source Augmentation (Restoration of Water Bodies)
• Source Augmentation (Groundwater)
• Major and Medium Irrigation - Supply Side Management
• Watershed Development - Supply Side Management
• Demand Side Management — Participatory Irrigation Practices
• Demand Side Management — Sustainable on-farm Water Use
Practices
• Rural Drinking Water
• Urban Water Supply and Sanitation
• Policy and Governance
The index would provide useful information for the States and also for
the concerned Central Ministries/Departments enabling them to
formulate and implement suitable strategies for better management of
water resources.
Q Source: http://niti.gov.in/content/composite-water-management-
index

60. With reference to carbon nanotubes, consider the following


statements.
1. These nanotubes can conduct heat and electricity better than
copper.
2. They have higher strength than steel, but also weigh
significantly more.
Which of the above is/are correct?
a) 1 only
b) 2 only
c) Both 1 and 2
d) None
Solution: a)
Justification: It is a tube-shaped material, made of carbon, having a
diameter measuring on the nanometer scale.
Statement 1: Researchers have developed thin carbon nanotube (CNT)
textiles that exhibit high electrical conductivity and a level of toughness
about 50 times higher than copper films, currently being used in
electronics.
Statement 2: CNT are at least 100 times stronger than steel, but only
one-sixth as heavy, so nanotube fibers could strengthen almost any
material.
Learning: The carbon nanotube sheets have significant importance for
the reliable operation of smart skin and flexible electronics, including
biological and structural health monitoring sensors.
Inorganic nanotubes have also been synthesized.
Q Source: https://www.sciencedaily.com/terms/carbon_nanotube.htm
61.The Prime Minister recently (2017) spoke of "Panch Tatva" in the
context of North-east India. He made a reference to Panch tatva in
the context of
a) Infrastructural Development of North-east
b) Contribution of North-eastern India to the Spiritual growth of
our nation
c) Border security from five vantage points
d) Safeguarding the rights of traditional forest dwelling
communities
Solution: a)
Learning: The PM recently laid the Foundation Stone of the IRAI near
Assam-Arunachal Pradesh border.
At the ceremony, he talked about Panch tatvas - five elements to boost
connectivity in the North-East: Railways, Highways, Airways,
Waterways, and i-ways.
He said his Government is working towards boosting prosperity and
welfare of the people in the North-East through this "panch-tatva."
Traditionally Pancha tatva is water, fire, ether, air and earth, as known in
the Indian vedic philosophy.
Q Source: http://pib.nic.in/newsite/PrintRelease.aspx?relid=162157

62. The 42nd Amendment to the Constitution amended the


Preamble to add the words
a) Sovereign and democratic
b) Socialist and Secular
c) Republic and Integral
d) Liberty and Equality
Solution: b)
Justification: It changed the description of India from "sovereign
democratic republic" to a "sovereign, socialist secular democratic
republic", and also changed the words "unity of the nation" to "unity and
integrity of the nation".
Learning: The 42nd amendment was a major as well as controversial
amendment to the constitution.
In Minerva Mills case, the Supreme Court declared unconstitutional two
provisions of the 42nd Amendment which prevent any constitutional
amendment from being "called in question in any Court on any ground"
and accord precedence to the Directive Principles of State Policy over the
Fundamental Rights of individuals respectively.
Q Source: Amendments to the Indian Constitution

63. The initiative RuTAG was started to


a) Give Geographical Indication (GI) tags to rural made products
b) Increase penetration of social media in Rural India
c) Identify budding intellects from small villages and towns of
India
d) Provide technical support to NGOs involved in rural
development
Solution: d)
Learning: RuTAG is supported by the office of Principal Scientific
Adviser to the Government of India.
Mandate of RuTAG:
• To promote traditional wisdom and knowledge in the field of rural-
technology using available human and natural resources.
• To maintain environmental sustainability and conserve bio-
diversity in rural area.
• To create appropriate and sustainable rural technology for the
critical needs of rural occupational groups
• To provide Coordination and support to science and technology by
NGO's.
• To established network with different centers of excellence,
research institutes and PSU's engaged in the field of rural
development in the areas.
Q Source: http://www.psa.gov.in/initiatives/concept-rutag

64. What are the advantages of Fertigation, a method of


application of fertilisers, as against conventional methods?
1. Nutrients and water are supplied near the active root zone
through fertigation.
2. Efficiency of fertilizer application is higher as compared to
conventional methods.
3. Urea and nitrogen-based fertilizers are incompatible with
fertigation, which reduces soil degradation that usually occurs
due to fertilizers.
4. Weed growth on the field can be controlled with fertigation.
Select the correct answer using the codes below.
a) 1 and 3 only
b) 2 and 4 only
c) 1, 2 and 4 only
d) 2, 3 and 4 only
Solution: c)
Justification: Fertigation is a method of fertilizer application in which
fertilizer is incorporated within the irrigation water by the drip system.
In this system fertilizer solution is distributed evenly in irrigation.
Statement 1 and 2: By this method, fertilizer use efficiency is increased
from 80 to 90 per cent as they are delivered directly to roots via drip. As
water and fertilizer are supplied evenly to all the crops through
fertigation there is possibility for getting 25-50 per cent higher yield.
Statement 3: Urea, potash and highly water soluble fertilizers are
available for applying through fertigation. In fact, Urea is well suited for
injection in micro irrigation system. It is highly soluble and dissolves in
non-ionic form, so that it does not react with other substances in the
water.
Statement 4: Drip irrigation also avoids water spillage on the field which
could have promoted weed growth. So, fertigation also helps control
weed growth.
Q Source:
http://agritech.tnau.ac.in/agriculture/agri_nutrientmgt_fertigation.htm
l

65. Ad valorem duties are based on


a) Value of goods
b) Dimension and quality of goods
c) A flat rate of taxation
d) Regional origin of goods
Solution: a)
Learning: A specific rate of duty is a flat rate, say Rs. 100 per packet
irrespective of the cost of the packet.
Ad valorem is based on the value, say 10% of the value of the packet.
A compound duty is a combination of both a specific rate of duty and an
ad valorem rate of duty.
Q Source: Related to Tax reforms in India

66. Consider the following statements about the fisheries sector


in India.
1. India is the leading producer of fisheries in the world.
2. Most of the marine fish production comes from the East coast.
Which of the above is/are correct?
a) 1 only
b) 2 only
c) Both 1 and 2
d) None
Solution: d)
Justification: Statement 1: India produces about 3 per cent of world’s
fish and occupies seventh place among the fish producing nations of the
world.
Fishing provides employment over one million people and account for
about one per cent of the total agricultural production in India.
Statement 2: The entire Indian coastline is suitable for marine fisheries.
But, it is estimated that about 75 per cent of the marine fish landings are
on the West coast and only 25 per cent is contributed by East coast.
Learning: These are some challenges to fish production in India:
• India has tropical climate in which fish cannot be preserved for a
long time. Heavy expenditure on refrigeration and deep freezing
increases the market price of the fish.
• Indian coast does not have many gulfs, bays, estuaries and
backwaters as is the case with Norway. As such, it lacks good
fishing grounds
• Marine fishing in India is a seasonal phenomena. Strong winds
during the monsoon season accompanied by tropical cyclones
often hinder the fishing operation.
Q Source: Human Geography: The Economy – by Pradeep Sharma

67. The Gandhi–Irwin Pact was signed in view to


a) Prepare a new constitution for India
b) End the laissez fairez trade policy of the British
c) Discontinue the Civil Disobedience Movement and change salt
laws
d) Declare the suzerainty of the British crown over India
Solution: c)
Learning: It was signed before the second Round Table Conference in
London.
Before this, the viceroy Lord Irwin announced in 1929, a vague offer of
'dominion status' for India in an unspecified future and a Round Table
Conference to discuss a future constitution.
The proposed conditions under the pact were:
• Discontinuation of the civil disobedience movement by the Indian
National Congress
• Participation by the Indian National Congress in the Round Table
Conference
• Withdrawal of all ordinances issued by the British Government
imposing curbs on the activities of the Indian National Congress
• Withdrawal of all prosecutions relating to several types of offenses
except those involving violence
• Release of prisoners arrested for participating in the civil
disobedience movement
• Removal of the tax on salt, which allowed the Indians to produce,
trade, and sell salt legally and for their own private use
Q Source: Spectrum Modern India by Rajiv Ahir

68. The UNEP-led Green Economy Initiative (GEI) includes


which of these activities?
1. Producing research materials analysing the implications of
green investment
2. Engaging non-governmental organizations, business and UN
partners in implementing Green Economy measures
3. Providing advisory services on ways to move towards a green
economy in specific countries
Select the correct answer using the codes below.
a) 2 only
b) 2 and 3 only
c) 1 only
d) 1, 2 and 3
Solution: d)
Justification: For the purposes of the Green Economy Initiative,
UNEP has developed a working definition of a green economy as one
that results in improved human well-being and social equity, while
significantly reducing environmental risks and ecological scarcities.
GEI was launched in late 2008 for promoting the Green Economy
Report and engaging partners in implementing this initiative.
UNEP believes that Green investments need to be catalyzed and
supported by targeted public expenditure.
Q Source:
http://staging.unep.org/greeneconomy/AboutGEI/WhatisGEI/tabid/29
784/Default.aspx

69. With reference to cut motions in the Parliament, consider the


following:
1. A departmental standing committee that examines the Budget
can suggest and examine cut motions.
2. Amendment of existing laws cannot be suggested by a cut
motion.
3. Cut motion cannot be raised on expenditure charged on the
Consolidated Fund of India.
Select the correct answer using the codes below.
a) 2 and 3 only
b) 1 only
c) 1 and 3 only
d) 2 only
Solution: a)
Justification: Statement 1: Such a standing committee can only
consider the demands for grants of the concerned ministries /
departments before they are discussed and voted in the Lok Sabha. Its
report should not suggest anything of the nature of cut motions.
Statement 2: A cut motion cannot raise policy matters. It can only
suggest reduction of unnecessary expenditures.
Statement 3: These expenditure are not to be voted upon by the
Parliament, hence exempt from cut motions.
Q Source: Chapter on Parliament: Indian Polity: M Laxmikanth

70. ECOLEX is a/an


a) Treaty against ecological patrolling in global commons
b) Index to measure species richness in an ecosystem
c) Satellite launched by UNFCCC to monitor GHG emissions
d) Information service on environmental law
Solution: d)
Learning: It is operated jointly by FAO, IUCN and UNEP.
Its purpose is to build capacity worldwide by providing the most
comprehensive possible global source of information on environmental
law .
The ECOLEX database includes information on treaties, international
soft-law and other non-binding policy and technical guidance
documents, national legislation, judicial decisions, and law and policy
literature. Users have direct access to the abstracts and indexing
information about each document, as well as to the full text of most of
the information provided.
Q Source: IUCN website

71. Cellulose is not a significant source of pure glucose, as compared to


starch, because
1. Very little cellulose is available for conversion compared to
starch.
2. Cellulose cannot be converted into Glucose other than by
degrading at high temperatures.
3. Cellulose resists biodegradation unlike starch.
Select the correct answer using the codes below.
a) 1 and 2 only
b) 3 only
c) 1 and 3 only
d) 2 only
Solution: b)
Justification: Statement 1: There is very much more cellulose
available as the biomass of plats, and is as a potential source of glucose,
than starch, yet it is not a significant source of pure glucose.
Statement 2: Currently, there are two major ways of converting cellulose
to glucose: chemical versus enzymatic. These are tricky.
However, starch is directly produced in relatively pure forms by plants
for use as an easily biodegradable energy and carbon store. This makes it
a preferable choice for glucose production.
Statement 3: On the other hand, Cellulose is structural and is
purposefully combined and associated with lignin and pentosans, so as
to resist biodegradation.
You can understand this by the fact that dead trees take several years to
decay even in tropical rainforests.
Owing to these reasons, research on both methods has for decades
occupied the attention of many investigators world wide.
Q Source: Improvisation: 12th Biology NCERT

72. With reference to Ancient India, consider the following:


1. Mandalam: A province
2. Valanadu: Grant to temples
3. Manigramam: Name of a merchant Guild
4. Ur: A district in poverty stricken areas
Which of the above are correctly matched?
a) 1 and 3 only
b) 2 and 4 only
c) 3 only
d) 1, 2, 3 and 4
Solution: a)
Justification: Statement 1 and 2: Each province was further sub-
divided into units like bhukti (under Guptas), rashtra (under
Rashtrakutas), mandala (under Cholas and Chalukyas) etc.
So, Mandalam stood for a province, and valanadu for a district.
Statement 3: These highly influential merchant guilds flourished in
Tamil Nadu in the Pallava and Chola periods.
Statement 4: In South India, there were two types of assemblies the Ur
and Sahba. While the former was the general type the latter one was
peculiar to the Brahmadeya villages.
Q Source: Medieval India – Romila Thapar: Class VII: Old NCERT

73. As per Mahatma Gandhi "God is truth” and “The way to truth
lies through” .…..?
a) Non-Violence
b) Resistance
c) Fasting
d) Love and devotion
Solution: a)
Learning: Mahatma Gandhi, proclaimed this in a note in English made
at Sabarmati Ashram in 1927.
• He also quoted at another place, “Ahimsa is the highest ideal. It is
meant for the brave, never for the cowardly. The highest religion
has been defined by a negative word: Ahimsa.
• These are excerpts from other notes, “Dharma is one and only one.
Ahimsa means moksha, and moksha is the realization of Truth.”
• “The most distinctive and largest contribution by Hinduism to
India’s culture is the doctrine of Ahimsa.”
Q Source: Quotes by Mahatma Gandhi

74. Why do stars twinkle but planets do not?


a) Light in stars is not produced by any chemical reaction as in
planets.
b) Stars are not at a constant distance with respect to earth, unlike
planets which maintain constant distance.
c) Planets are nearer due to which light does not refract much
from the atmosphere
d) There are no gases found in any of the stars nearby to earth.
Solution: c)
Learning: Planets are also closer to the Earth than those distant stars,
so planets appear larger in comparison.
Due to the planets' apparent closeness to Earth, the light coming from
these celestial bodies does not bend much due to Earth's atmosphere.
They appear as point sources even when viewed by telescopes. Since the
Earth's atmosphere is turbulent, all images viewed up through it tend to
"swim." The result of this is that sometimes a single point in object
space gets mapped to two or more points in image space. This shows as
sparkling.

Q Source: 10th Science NCERT

75. Agriculture Ministry has issued detailed guidelines and


strategy to be adopted for implementation of the National Mission
on Oilseeds and Oil Palm (NMOOP). The strategy includes
1. Reducing Seed Replacement Ratio (SRR) to zero
2. Diversification of area from low yielding cereals crops to
oilseeds crops
3. Processing of tree borne oilseeds
Select the correct answer using the codes below.
a) 1 and 2 only
b) 3 only
c) 2 and 3 only
d) 1, 2 and 3
Solution: c)
Background: There are two kinds of seeds – farm saved seeds and
certified seeds.
• A farmer can grow his crop by using either of them.
• Seed Replacement Ratio is a measure of how much of the total
cropped area was sown with certified seeds in comparison to farm
saved seeds.
• Since certified seeds have higher productivity, farmers prefer them,
and a higher SRR directly translates into higher productivity.
Justification: Statement 1: The strategy includes increasing Seed
Replacement Ratio (SRR) with focus on Varietal Replacement so as to
increase yield.
Statement 2: It also focuses on increasing irrigation coverage under
oilseeds from 26% to 36%; diversification of area from low yielding
cereals crops to oilseeds crops; inter-cropping of oilseeds with cereals/
pulses/ sugarcane; and use of fallow land after paddy /potato cultivation.
Statement 3: The scheme aims at enhancing production of traditional
oilseed and tree-borne oilseed. In addition, significant area is sought to
be brought under oil palm.
Inter-cropping during gestation period of oil palm and tree borne
oilseeds would provide economic return to the farmers when there is no
production.
Q Source: http://nmoop.gov.in/Guidelines.aspx

76. LOTUS HR project of the Government of India deals with


a) Increasing participation of women in positions of authority and
leadership
b) Declaration of additional National Symbols
c) Conservation of keystone plant species
d) Urban sewage treatment
Solution: d)
Learning: The project LOcal Treatment of Urban Sewage Streams for
Healthy Reuse (LOTUS HR) is an Indo-Netherlands joint project funded
by Department of Biotechnology (DBT), Govt of india and NWO/STW,
The Govt of Netherlands.
• The project has stakeholders from both Academia and Industry
from India and Netherlands, coordinated by IIT Delhi and TERI.
• The project aims to demonstrate a novel holistic waste-water
management approach, that will produce clean water that can be
reused for various proposes (e.g. industry, agriculture,
construction etc.), while simultaneously recovering nutrients and
energy from the urban waste water.
• This will help convert drains into profitable mines. Special
attention will be paid to pathogen removal and removing
conventional and emerging pollutants.
Q Source: PIB: RDS/nb: Release Id :161675

77.Consider the following statements about some organizations


abroad:
1. Paris Indian Society was an Indian nationalist organization
founded under the patronage of Madam Bhikaji Cama.
2. India House received the patronage of Shyamji Krishna Varma
and was opened to promote nationalist views among Indian
students in Britain.
Which of the above is/are correct?
a) 1 only
b) 2 only
c) Both 1 and 2
d) None
Solution: c)
Justification: Statement 1: The organisation was opened as a branch of
the Indian Home Rule Society founded that same year in London under
the patronage of Shyamji Krishna Varma.
• The Paris Indian Society also saw active participation from Indian
nationalists who at various times were associated with the India
House during its short existence.
• Following the liquidation of the India House in the wake of Curson
Wyllie's assassination in 1909 by Madanlal Dhingra, the PIS
became the refuge and hub of Indian revolutionaries who fled
England
Statement 2: India House was a student residence that existed between
1905 and 1910 in London.
• The building rapidly became a hub for political activism, one of the
most prominent for overseas revolutionary Indian nationalism.
• Patrons of India House published an anti-colonialist newspaper,
The Indian Sociologist, which the British Raj banned as "seditious"
Q Source: Spectrum Modern India by Rajiv Ahir

78. With reference to the International Energy Charter, consider


the following:
1. It is a declaration of political intention aiming at strengthening
energy cooperation between the signatory states.
2. It is legally binding on member states and involves annual
financial commitment to the corpus of the charter.
3. India recently agreed to sign and ratify the charter.
Select the correct answer using the codes below.
a) 2 only
b) 1 and 3 only
c) 1 only
d) 2 and 3 only
Solution: c)
Background: The Energy Charter Treaty (ECT) is an international
agreement which establishes a multilateral framework for cross-border
cooperation in the energy industry. It strives to promote principles of
openness of global energy markets and non-discrimination to stimulate
foreign direct investments and global cross-border trade.
Justification: Statement 1: In 2015, a major milestone in
modernisation and expansion of the Energy Charter was reached, as 72
Countries plus the EU signed the International Energy Charter.
The International Energy Charter is a political declaration about
principles for international energy cooperation.
Statement 2: No such commitments are provided for, however it is
intended as a first step towards accession to the legally binding Energy
Charter Treaty.
Statement 3: The treaty covers all aspects of commercial energy
activities including trade, transit, investments and energy efficiency.
India is not a signatory.
Q Source: http://www.energycharter.org/what-we-
do/conexo/overview/

79. Consider the following statements.


1. Nuclear medicine uses trace amounts of radioactive substances
in the diagnosis and treatment of diseases.
2. Radio therapy uses high energy particles at tumours or specific
parts of the body to destroy the malignant cells.
Which of the above is/are correct?
a) 1 only
b) 2 only
c) Both 1 and 2
d) None
Solution: c)
Justification: Statement 1: Nuclear medicine imaging uses small
amounts of radioactive materials called radiotracers that are typically
injected into the bloodstream, inhaled or swallowed.
• The radiotracer travels through the area being examined and gives
off energy in the form of gamma rays which are detected by a
special camera and a computer to create images of the inside of
your body.
• Nuclear medicine has proven its worth in the diagnosis of diseases
such as cancer, Alzheimer’s etc. in their initial stages permitting
early treatment as well as reduced morbidity and mortality.
Statement 2: Your cells normally grow and divide to form new cells. But
cancer cells grow and divide faster than most normal cells.
• Radiation works by making small breaks in the DNA inside cells.
• These breaks keep cancer cells from growing and dividing and
cause them to die.
• Nearby normal cells can also be affected by radiation, but most
recover and go back to working the way they should.
• Unlike chemotherapy, which usually exposes the whole body to
cancer-fighting drugs, radiation therapy is usually a local
treatment.
Q Source: http://www.dbtindia.nic.in/program-medical-
biotechnology/

80. Amuktamalyada was a/an


a) Long poem in Telugu by Krishnadeva raya in which he explains
how a King should be
b) Story of a Pallava ruler who sacrificed his kingdom to win back
his subjects
c) Account of the ruling history of the Mauryas written by
Chandragupta Maurya himself
d) Segment of Arthashashtra that became popular for its dictats on
foreign policy
Solution: a)
Learning: Amuktamalyada describes the story of wedding of the Hindu
Lord Vishnu and Andal the Tamil Alvar poet and daughter of Periyalvar,
at Srirangam.
It is basically an epic poem in Telugu composed by Krishnadevaraya of
the Vijayanagar Dynasty.
Krishnadevaraya was a great patron of arts and a great scholar himself.
Q Source: Medieval India – Romila Thapar: Class VII: Old NCERT

81. Which of these best characterizes the nature of the Annual


Financial Statement or the Union Budget before it is presented to
the Parliament for the first time in the year?
a) It is a public document and the draft is shared with major civil
society bodies.
b) It is a secret document until it is revealed first to the Parliament.
c) It is disclosed to select corporate before unveiling to the
Parliament.
d) It is circulated at least a month before to all individual MPs
before being presented in the parliament on the budget day.
Solution: b)
Learning: Budget is always kept as a secret, otherwise those who have
prior information of the provisions can gain a lot from information
asymmetry.
For e.g. government may plan to build a railway station in a remote area
which would lead to shooting land prices; if some know this provision,
they will invest beforehand and exploit the information asymmetry.
When T.T.Krishnamachari was the Finance Minister, he actually had to
resign because the budget was leaked before presenting in the
Parliament.
Q Source: General questions on the Budget

82. SBI Blue Chip Fund is a mutual fund. What does the “Blue
Chip” in the fund name denote?
a) Sea-faring companies
b) Reputed firms able to absorb economic downturns
c) Emerging start-ups
d) Semi-conductor manufacturing companies
Solution: b)
Learning: A blue chip is stock in a corporation with a national
reputation for quality, reliability, and the ability to operate profitably in
good times and bad.
Blue chip stocks are seen as a less volatile investment than owning
shares in companies without blue chip status because blue chips have an
institutional status in the economy.
The name "blue chip" came about because in the game of poker the blue
chips have the highest value.
Q Source: Terminologies in news

83. On visiting a field, you would recommend Mixed cropping to


which of these farmers?
a) The one sowing crops with varying maturity periods
b) The one sowing crops with different nutritional requirements
c) The one who is sowing crops on different soils
d) The one who is not using any natural or chemical pesticides or
weedicides in his farm
Solution: b)
Justification: This type of cropping leads to an improvement in the
fertility of the soil and hence, increase in crop yield because when the
two crops are properly chosen the products and refuse from one crop
plant help in the growth of the other crop plant and vice-versa.
Mixed cropping is an insurance against crop failure due to abnormal
weather conditions. Some successful mixed cropping practices are:
Soyabean + Pigeon pea; Maize + udad dal (Black gram); Pigeon pea +
Mung dal (Green gram); Groundnut + Sunflower etc.
Q Source: 9th Geography NCERT

84. When a voter presses a button in the Electronic Voting


Machine (EVM), a paper slip is printed through the VVPAT. This
slip contains the
1. Name and caste of the Voter
2. Poll symbol of the voted candidate
3. Name of the voted candidate
4. Address where the EVM is installed
Select the correct answer using the codes below.
a) 1 and 3 only
b) 2 and 4 only
c) 2 and 3 only
d) 2, 3 and 4 only
Solution: c)
Justification: The Voter Verified Paper Audit Trail is attached to
electronic voting machines.
This slip contains the poll symbol, serial number and name of the
candidate only. It allows the voter to verify his/her choice.
After being visible to the voter from a glass case in the VVPAT for seven
seconds, the ballot slip will be cut and dropped into the drop box in the
VVPAT machine and a beep will be heard. VVPAT machines can be
accessed by polling officers only.
Questions on VVPAT have been covered in Revision tests and Insights
Daily quizzes. If you want to learn more about it, go to the Q Source.
Q Source: http://www.thehindu.com/news/national/all-you-need-to-
know-about-vvpat/article18077550.ece
85. Which of these will have a bearing on increasing the pulse
production in India?
1. Successful implementation of National Food Security Mission
2. Bringing Green Revolution to Eastern Areas
3. Increased Inter-cropping of pulses with suitable crops
Select the correct answer using the codes below.
a) 2 only
b) 1 and 3 only
c) 3 only
d) 1, 2 and 3
Solution: d)
*Justification*: Statement 1: NFSM focuses on pulses, cereals, millets
and selected commercial crops. But, a decision was taken to exclusively
dedicate 50% allocations in NFSM for development of pulses alone.
Statement 2 and 3: Cultivation of pulses is being promoted as an
intercrop with cereals, oilseeds and cash crops with introduction of
suitable varieties and package of practices.
• Scientists have demonstrated successful cultivation of pigeon pea
on bunds of rice fields.
• Similarly, pulse crop is being encouraged in rice fallow areas under
‘Bringing Green Revolution in Eastern India’ scheme.
• Rice fallows in Gangetic plains offer a huge potential for expansion
of the area of rabi pulses such as chickpeas and lentils.
Q Source: Kurukshetra: November 2016

86. This newly discovered plant with pink coloured flowers is a


Critically Endangered species as enlisted by IUCN. Each plant
carries just two flowers and it grows in soil found in the gaps of
rocks. It was found in India in
a) Gopalpur on Sea
b) Drylands of Malwa
c) Forests of Western Ghats
d) Marshes of Kutch
Solution: c)
Learning: This discovery from biodiversity rich Nelliampathy (Western
Ghats) has been published in the latest issue of Phytotaxa , an
international journal on taxonomy.
Interestingly, the researchers have named the plant as Sonerila nairii as
a tribute to an eminent Indian botanist P.K.K. Nair.
The plant species is lithophytic (grow on rocks) and flowers during the
October-November months in climatic regions with altitudes ranging
above 1,200 metres above sea level.

Q Source: http://www.thehindu.com/news/national/kerala/new-
plant-species-discovered/article8411662.ece
87. Archaeological evidences of Painted Grey Ware (PGW)
CANNOT be found in which of these regions?
a) Panipat
b) Hastinapura
c) Kurukshetra
d) Nigwaram
Solution: d)
Learning: Excavations at places for e.g. Hastinapura, Kurukshetra,
Panipat, Tilpat, Baghpat, Mathura, and Bairat—have given evidence of a
pottery Painted Grey Ware (PGW) which goes back to c. 1000 BCE.
This shows that these sites were inhabited around this time, and the
nature of the remains suggests that the people who lived here shared a
pastoral-cum-agricultural lifestyle.
Excavations at Hastinapura also give evidence of a flood in the Ganga,
after which the site was deserted
Q Source: Upinder Singh: A History of Ancient and Early Medieval
India

88. Consider the following about Pradhan Mantri Surakshit


Matritva Abhiyan (PMSMA).
1. It aims to provide free health check-ups to pregnant women at
government health centres and hospitals.
2. It provides post-natal financial support system to women by
transferring them a lump sum amount every year for child
protection.
Which of the above is/are correct?
a) 1 only
b) 2 only
c) Both 1 and 2
d) None
Solution: a)
Justification: The national programme aims to provide pregnant
ladies free ante-natal services (ANC) and required treatment for free
every month.
The scheme is applicable only for the pregnant women in their
pregnancy period of 3 to 6 months. It will provide all kinds of medical
check-ups completely free to pregnant women.
These check-ups will take place at the medical centres, government and
private hospitals and private clinics across the country.
It is aimed to reduce maternal and infant mortality rates through safe
pregnancies and safe deliveries.
Q Source: Government Schemes

89. CARE International is a non-governmental organization


committed to
a) Fighting poverty and empowering poor
b) Lobbying with governments to strengthen democracy and its
institutions
c) Reconstructing ancient International pathways
d) Stopping infiltration of arms in war prone zones
Solution: a)
Learning: They focus in empowering poor women because women have
the power to help whole families and entire communities escape poverty.
They also work towards strengthening capacity for self-help; providing
economic opportunity; delivering relief in emergencies; influencing
policy decisions at all levels and addressing discrimination in all its
forms.
For e.g. CARE is providing long-term help to thousands of women
struggling to raise their families after years of sectarian violence in Iraq.
Q Source: Major International NGOs
90. The Global Taxonomy Initiative (GTI) was established by
a) Alliance for Zero Extinction
b) Conference of Parties (COP) to the Convention on Biological
Diversity (CBD)
c) International Union for Conservation of Nature (IUCN)
d) Greenpeace International
Solution: b)
Learning: It was established in 1998 by CoP.
• Taxonomy is the science of naming, describing and classifying
organisms and includes all plants, animals and microorganisms of
the world.
• Unfortunately, taxonomic knowledge is far from complete.
• Taxonomy provides basic understanding about the components of
biodiversity which is necessary for effective decision-making about
conservation and sustainable use.
• It is also essential to combat Invasive Alien Species.
Q Source: https://www.cbd.int/gti/default.shtml

91.A capitalist economy features


a) Nationalization of all property
b) Non-existence of competitive markets
c) Market based pricing of products
d) Public provision of all goods and services
Solution: c)
Learning: It is based on private ownership of the means of production
and their operation for profit.
Characteristics central to capitalism include private property, capital
accumulation, wage labor, voluntary exchange, a price system, and
competitive markets.
Q Source: 12th Macroeconomics NCERT
92. The SAARC country which has a common border with
another ‘member country’ other than India is
1. Sri Lanka
2. Maldives
3. Bhutan
4. Pakistan
Select the correct answer using the codes below.
a) 2 and 3 only
b) 1 and 4 only
c) 3 and 4 only
d) None of the above
Solution: d)
Justification: No SAARC country other than India has a common
border with another member country, Sri Lanka and Maldives being
islands
Moreover, it is pertinent to remember that NOT all countries with which
India has border are members of SAARC.
Moreover, not all the SAARC countries have common border with China
Q Source: Map based questions

93. Which of these nationalist writers used literature, poetry and


speech as a tool for political and social awareness during the
Indian Independence Movement?
1. Subramaniya Bharathi
2. Bankim Chandra Chattopadhyay
3. Begum Rokeya
Select the correct answer using the codes below.
a) 1 only
b) 1 and 3 only
c) 2 only
d) 1, 2 and 3
Solution: d)
Justification: Statement 1: Popularly known as "Mahakavi Bharati",
he was a pioneer of modern Tamil poetry and is considered one of the
greatest Tamil literary figures of all time.
Statement 3: She was a Bengali writer who became famous for her work
– Sultana’s Dream – a science fiction novella set in a place called Lady
Land, a world ruled by women.
Learning: Other such writers and poets were Allama Iqbal, Josh
Malihabadi, Mohammad Ali Jouhar, and Kazi Nazrul Islam.
Q Source: Indian Independence Movement: Wikipedia

94. The Bonn Challenge is a global effort to


a) Reduce crime rates in both developed and developing
economies to ten per cent of existing rates
b) Boost international tourism in Least Developed Countries
(LDCs) within limits of sustainable development
c) Bring millions of hectares of degraded and deforested land into
restoration
d) Finance the education of every girl child on this planet by 2030
Solution: c)
Learning: It aims to bring 150 million hectares of degraded and
deforested land into restoration by 2020 and 350 million by 2030.
• The Global Environment Facility (GEF) Council has approved The
Restoration Initiative (TRI) that will help 10 countries define and
achieve commitments under the Bonn Challenge.
• IUCN (as lead), FAO and UNEP will jointly implement TRI.
• New York Declaration on Forests of the 2014 UN Climate Summit
is associated with this challenge.
Q Source: GEF Website
95. Subhash Chandra Bose resigned as the President of Indian
National Congress (INC) at the Tripuri Session mainly due to
a) Differences in opinion with Gandhi
b) INC supporting the forced entry of India in World War II
c) Suspension of non-cooperation movement due to Chauri
Chaura Incident
d) Lucknow pact signed by the INC with the Muslim League
Solution: a)
*Justification*: Option B can be eliminated as INC never supported
entry of India in WWII. It opposed it vehemently and even resigned from
several provinces in 1939.
Option C and D can be eliminated as Subhash Chandra Bose became the
President of INC in 1938, and these events in C and D occurred much
before.
Learning: For the 1939 elections of the President of Congress, Subhash
announced his candidature.
• Gandhi suggested Nehru to announce his name as a candidate.
• But Dr. Pattabhi Sitaramayya, an Andhra leader was suggested for
candidature against Subhash.
• In the INC elections, however, Subhash won, and this win of
Subhash was taken by Gandhi as a personal blow.
• This started conflicts between Subhash and Gandhi who took
election of Subhash a threat to the ideology of INC.
• Due to the steep differences, Subhash resigned from INC and
formed All India Forward Bloc. This is known as the Tripuri crisis
in the INC.
Q Source: Spectrum Modern India by Rajiv Ahir

96. Conservation International (CI) is a non-profit organization


that
1. Directly works with national governments around the world to
identify and design projects related to environmental
conservation
2. Works as an accredited project agency of the Global
Environment Facility (GEF) disbursing funding for conservation
and sustainable development initiatives
Which of the above is/are correct?
a) 1 only
b) 2 only
c) Both 1 and 2
d) None
Solution: c)
Justification: The GEF is the largest public funder of projects to
improve the global environment.
• The GEF channels its funding through GEF agencies, which are
responsible for assisting eligible governments and nonprofit
organizations in the development, implementation and
management of GEF projects.
• In 2013, Conservation International (CI) became one of the first
nonprofit organizations to be accredited as a GEF Project Agency.
• As such, CI directly works with national governments around the
world to identify and design projects and advise on project
execution, making sure that projects meet high technical and
financial standards, while complying with the GEF’s environmental
and social safeguards.
Q Source: http://www.conservation.org/gef/Pages/default.aspx

97. Consider the following about Carbon Offsetting and


Reduction Scheme for International Aviation (CORSIA) developed
by the International Civil Aviation Organization (ICAO).
1. ICAO decided to develop CORSIA because aviation emissions
were not covered under Paris Agreement.
2. India led the negotiation committee that drafted CORSIA and
recently ratified the scheme that will be operational from 2018.
Which of the above is/are correct?
a) 1 only
b) 2 only
c) Both 1 and 2
d) None
Solution: d)
Justification: Aviation (domestic and international) accounts for
approximately 2 per cent of global CO2 emissions produced by human
activity. This led ICAO to develop CORSIA.
The CORSIA calls for international aviation to address and offset its
emissions through the reduction of emissions elsewhere (outside of the
international aviation sector), involving the concept of "emissions units".
This is similar to sale of carbon trading units.
Statement 1: Emissions from domestic aviation are considered under
the UNFCCC. Therefore, Parties to the UNFCCC have the possibility of
addressing these emissions in their Nationally Determined
Contributions (NDCs) under the Paris Agreement. Statement 1 is wrong.
Agreement on the CORSIA contributes to the level of ambition set by the
Paris Agreement.
Statement 2: India hasn’t joined the accord, and this has disappointed
IATA. But, it is hope that India will join soon as it has a large fleet of
flight operators and has led the Paris and Kigali agreements from the
forefront.
Q Source: http://www.business-standard.com/article/pti-
stories/india-keeping-away-from-corsia-a-major-concern-iata-
116122500283_1.html

98. With reference to the digital campaigns of the government,


which of the following statements is INCORRECT?
a) Digi Dhan Abhiyaan was initiated to promote digital payments
across the country.
b) DigiGaon initiative will provide tele-medicine, education and
skills through use of digital technologies in rural areas.
c) BHIM App is designed to keep mobile phones secure from
major cyber attacks.
d) MyGov is an online platform created for participative
governance.
Solution: c)
Learning: Option A: Digi Dhan Abhiyaan was launched in 2016. Under
this, more than two Crore people and over seven lakh shopkeepers were
trained in digital payments across 640 districts.
Option B: DigiGaon is a part of Digital Literacy Campaign where
providing telemedicine, education and skills by using digital technology
is proposed to improve access to skill programme and enhance
employment options.
Option C: BHIM App will catalyse the power of mobile phones for digital
payments and financial inclusion. It is a payments app that does not
require account number for every transfer or transaction.
Option D: It is a website where you can share ideas, collaborate with
like-minded people and directly help improve governance.
Q Source: Yojana – March 2017 and
http://pib.nic.in/newsite/PrintRelease.aspx?relid=158376 (The link is a
jist of all digital programmes of the government)

99. M.K. Gandhi was the editor of three English weeklies, which
of these was NOT one of them?
a) Harijan
b) Young India
c) Indian Opinion
d) Statesman
Solution: d)
*Learning*: Statesman is related to Robert Knight, not Gandhi.
Gandhi edited Indian Opinion (in South Africa during 1903-1915), Young
India (1919- 1931), and Harijan (1933-1942 and 1946-January 1948).
Indian Opinion was bi-lingual (English and Gujarati). For some time it
had also Hindi and Tamil sections.
Young India had a Gujarati edition - Navajivan. Harijan had both
Gujarati and Hindi editions.
All these journals which he described as "newspapers" were organs of
political and social movements and discussed with intensity and
concentration, problems that demanded immediate action.
Q Source: Publications by Gandhi

100. Consider the following:


Vanjeevan, established with the assistance of United Nations
Development Programme (UNDP), under will
1. Act as research and technical hub to further socio-economic
development of tribal communities
2. Promote and develop sustainable livelihood avenues in the
tribal areas through entrepreneurship and skill up-gradation
Which of the above is/are correct?
a) 1 only
b) 2 only
c) Both 1 and 2
d) None
Solution: c)
Justification: It is the National Resource Centre on Tribal Livelihood
(Vanjeevan).
• NRC is housed in NSTFDC (National Scheduled Tribes Finance
and Development Corporation) with UNDP assistance.
• Vanjeevan will be a programme to identify the problems in
livelihood issues in select districts of six states having low HDI of
tribal people in the first phase.
• Vanjeevan will also serve as an apex central institution within
Ministry of Tribal Affairs to act as research and technical hub to
further socio-economic development of tribal communities
through comprehensive interaction of the 3Es - employment,
employability and entrepreneurship.
• Vanjeevan will forge linkages and build upon the skill building
efforts of the other Central Ministries/ Departments such as
MoSDE, MoRD, MoMSME etc.
• The programme will focus on the identification of local resources,
keeping in view the existing skill level.
Q Source: PIB: Release Id :155295

S-ar putea să vă placă și